Produce formula












1














Let $a_{i,j}$ any positive integers wher $1leq i,j leq 3$.



I want to write closed form for following summation:



$$a_{1,3}(a_{2,1}+a_{2, 2}+a_{2,3})(a_{3,1}+a_{3, 2}+a_{3,3})+a_{2,3}(a_{1,1}+a_{1, 2})(a_{3,1}+a_{3, 2}+a_{3,3})+a_{3,3}(a_{1,1}+a_{1, 2})(a_{2,1}+a_{2, 2})$$



$textbf{My attempt:}$
begin{equation*}
sum_{j=1}^3 a_{j,3}left[prod_{substack{i=1 \ ineq j}}^3left(sum_{substack{k=1 \ k<j}}^{3 text{or} 3-1} a_{i,k} right) right]
end{equation*}

We will take $3-1$ when $k<j$.





Actually, I want to generalize this. My attempt so complicated to me.



If we consider all of $a_{i,j}$ as a matrix:
begin{pmatrix}
a_{1,1} & a_{1,2} & a_{1,3} \
a_{2,1} & a_{2,2} & a_{2,3} \
a_{3,1} & a_{3,2} & a_{3,3} \
end{pmatrix}

then are there any computer program to make formula?





$$a_{1,3}(a_{2,1}+a_{2, 2}+ a_{2,3})(a_{3,1}+a_{3, 2}+a_{3,3})cdots (a_{r,1}+a_{r,2}+a_{r,3})+a_{2,3}(a_{1,1}+a_{1, 2})(a_{3,1}+a_{3, 2}+a_{3,3})+cdots (a_{r,1}+a_{r,2}+a_{r,3})+a_{3,3}(a_{1,1}+a_{1, 2})(a_{2,1}+a_{2, 2})(a_{4,1}+a_{4,2}+a_{4,3}+cdots (a_{r,1}+a_{r,2}+a_{r,3}) \ vdots \
+a_{r,3}(a_{1,1}+a_{1,2})(a_{2,1}+a_{2,2})cdots (a_{r-1,1}+a_{r-1,2})
$$










share|cite|improve this question





























    1














    Let $a_{i,j}$ any positive integers wher $1leq i,j leq 3$.



    I want to write closed form for following summation:



    $$a_{1,3}(a_{2,1}+a_{2, 2}+a_{2,3})(a_{3,1}+a_{3, 2}+a_{3,3})+a_{2,3}(a_{1,1}+a_{1, 2})(a_{3,1}+a_{3, 2}+a_{3,3})+a_{3,3}(a_{1,1}+a_{1, 2})(a_{2,1}+a_{2, 2})$$



    $textbf{My attempt:}$
    begin{equation*}
    sum_{j=1}^3 a_{j,3}left[prod_{substack{i=1 \ ineq j}}^3left(sum_{substack{k=1 \ k<j}}^{3 text{or} 3-1} a_{i,k} right) right]
    end{equation*}

    We will take $3-1$ when $k<j$.





    Actually, I want to generalize this. My attempt so complicated to me.



    If we consider all of $a_{i,j}$ as a matrix:
    begin{pmatrix}
    a_{1,1} & a_{1,2} & a_{1,3} \
    a_{2,1} & a_{2,2} & a_{2,3} \
    a_{3,1} & a_{3,2} & a_{3,3} \
    end{pmatrix}

    then are there any computer program to make formula?





    $$a_{1,3}(a_{2,1}+a_{2, 2}+ a_{2,3})(a_{3,1}+a_{3, 2}+a_{3,3})cdots (a_{r,1}+a_{r,2}+a_{r,3})+a_{2,3}(a_{1,1}+a_{1, 2})(a_{3,1}+a_{3, 2}+a_{3,3})+cdots (a_{r,1}+a_{r,2}+a_{r,3})+a_{3,3}(a_{1,1}+a_{1, 2})(a_{2,1}+a_{2, 2})(a_{4,1}+a_{4,2}+a_{4,3}+cdots (a_{r,1}+a_{r,2}+a_{r,3}) \ vdots \
    +a_{r,3}(a_{1,1}+a_{1,2})(a_{2,1}+a_{2,2})cdots (a_{r-1,1}+a_{r-1,2})
    $$










    share|cite|improve this question



























      1












      1








      1







      Let $a_{i,j}$ any positive integers wher $1leq i,j leq 3$.



      I want to write closed form for following summation:



      $$a_{1,3}(a_{2,1}+a_{2, 2}+a_{2,3})(a_{3,1}+a_{3, 2}+a_{3,3})+a_{2,3}(a_{1,1}+a_{1, 2})(a_{3,1}+a_{3, 2}+a_{3,3})+a_{3,3}(a_{1,1}+a_{1, 2})(a_{2,1}+a_{2, 2})$$



      $textbf{My attempt:}$
      begin{equation*}
      sum_{j=1}^3 a_{j,3}left[prod_{substack{i=1 \ ineq j}}^3left(sum_{substack{k=1 \ k<j}}^{3 text{or} 3-1} a_{i,k} right) right]
      end{equation*}

      We will take $3-1$ when $k<j$.





      Actually, I want to generalize this. My attempt so complicated to me.



      If we consider all of $a_{i,j}$ as a matrix:
      begin{pmatrix}
      a_{1,1} & a_{1,2} & a_{1,3} \
      a_{2,1} & a_{2,2} & a_{2,3} \
      a_{3,1} & a_{3,2} & a_{3,3} \
      end{pmatrix}

      then are there any computer program to make formula?





      $$a_{1,3}(a_{2,1}+a_{2, 2}+ a_{2,3})(a_{3,1}+a_{3, 2}+a_{3,3})cdots (a_{r,1}+a_{r,2}+a_{r,3})+a_{2,3}(a_{1,1}+a_{1, 2})(a_{3,1}+a_{3, 2}+a_{3,3})+cdots (a_{r,1}+a_{r,2}+a_{r,3})+a_{3,3}(a_{1,1}+a_{1, 2})(a_{2,1}+a_{2, 2})(a_{4,1}+a_{4,2}+a_{4,3}+cdots (a_{r,1}+a_{r,2}+a_{r,3}) \ vdots \
      +a_{r,3}(a_{1,1}+a_{1,2})(a_{2,1}+a_{2,2})cdots (a_{r-1,1}+a_{r-1,2})
      $$










      share|cite|improve this question















      Let $a_{i,j}$ any positive integers wher $1leq i,j leq 3$.



      I want to write closed form for following summation:



      $$a_{1,3}(a_{2,1}+a_{2, 2}+a_{2,3})(a_{3,1}+a_{3, 2}+a_{3,3})+a_{2,3}(a_{1,1}+a_{1, 2})(a_{3,1}+a_{3, 2}+a_{3,3})+a_{3,3}(a_{1,1}+a_{1, 2})(a_{2,1}+a_{2, 2})$$



      $textbf{My attempt:}$
      begin{equation*}
      sum_{j=1}^3 a_{j,3}left[prod_{substack{i=1 \ ineq j}}^3left(sum_{substack{k=1 \ k<j}}^{3 text{or} 3-1} a_{i,k} right) right]
      end{equation*}

      We will take $3-1$ when $k<j$.





      Actually, I want to generalize this. My attempt so complicated to me.



      If we consider all of $a_{i,j}$ as a matrix:
      begin{pmatrix}
      a_{1,1} & a_{1,2} & a_{1,3} \
      a_{2,1} & a_{2,2} & a_{2,3} \
      a_{3,1} & a_{3,2} & a_{3,3} \
      end{pmatrix}

      then are there any computer program to make formula?





      $$a_{1,3}(a_{2,1}+a_{2, 2}+ a_{2,3})(a_{3,1}+a_{3, 2}+a_{3,3})cdots (a_{r,1}+a_{r,2}+a_{r,3})+a_{2,3}(a_{1,1}+a_{1, 2})(a_{3,1}+a_{3, 2}+a_{3,3})+cdots (a_{r,1}+a_{r,2}+a_{r,3})+a_{3,3}(a_{1,1}+a_{1, 2})(a_{2,1}+a_{2, 2})(a_{4,1}+a_{4,2}+a_{4,3}+cdots (a_{r,1}+a_{r,2}+a_{r,3}) \ vdots \
      +a_{r,3}(a_{1,1}+a_{1,2})(a_{2,1}+a_{2,2})cdots (a_{r-1,1}+a_{r-1,2})
      $$







      combinatorics combinations






      share|cite|improve this question















      share|cite|improve this question













      share|cite|improve this question




      share|cite|improve this question








      edited Nov 27 '18 at 13:23







      1Spectre1

















      asked Nov 27 '18 at 10:57









      1Spectre11Spectre1

      999




      999






















          1 Answer
          1






          active

          oldest

          votes


















          3














          $$a_{1,3}(a_{2,1}+a_{2, 2}+a_{2,3})(a_{3,1}+a_{3, 2}+a_{3,3})+a_{2,3}(a_{1,1}+a_{1, 2})(a_{3,1}+a_{3, 2}+a_{3,3})+a_{3,3}(a_{1,1}+a_{1, 2})(a_{2,1}+a_{2, 2})$$
          $$=(a_{1,1}+a_{1,2}+a_{1,3})(a_{2,1}+a_{2,2}+a_{2,3})(a_{3,1}+a_{3,2}+a_{3,3})-(a_{1,1}+a_{1,2})(a_{2,1}+a_{2,2})(a_{3,1}+a_{3,2})$$
          $$=prod_{i=1}^3sum_{j=1}^3a_{i,j}-prod_{i=1}^3sum_{j=1}^2a_{i,j}.$$
          More generally,
          $$sum_{i=1}^ra_{i,m}left(prod_{h=1}^{i-1}sum_{j=1}^{m-1}a_{h,j}right)left(prod_{h=i+1}^rsum_{j=1}^ma_{h,j}right)=prod_{i=1}^rsum_{j=1}^ma_{i,j}-prod_{i=1}^rsum_{j=1}^{m-1}a_{i,j}$$
          $$=sum_{min{j_1,j_2,dots,j_r}}a_{1,j_1}a_{2,j_2}cdots a_{r,j_r}.$$






          share|cite|improve this answer























          • Thank you for the answer. Is it possible to generalize it? I edited my question.
            – 1Spectre1
            Nov 27 '18 at 13:24






          • 1




            @1ENİGMA1 I cleaned up my answer.
            – bof
            Nov 28 '18 at 23:13






          • 1




            For example, $n!=prod_{i=1}^ni$ and in particular $0!=prod_{i=1}^0i=1$. Likewise, $x^n=prod_{i=1}^nx$, in particular, $x^0=prod_{i=1}^0x=1$.
            – bof
            Nov 29 '18 at 8:59








          • 1




            $sum_{min{j_1,j_2,dots,j_r}}a_{1,j_1}a_{2,j_2}cdots a_{r,j_r}$ is the sum of all products of the form $a_{1,j_1}a_{2,j_2}cdots a_{r,j_r}$ where at least one of the indices $j_1,j_2,dots,j_r$ is equal to $m$, or in other words, $m$ is an element of the set ${j_1,j_2,dots,j_r}$.
            – bof
            Nov 29 '18 at 9:04






          • 1




            $$prod_{i=1}^2sum_{j=1}^2a_{i,j}=(a_{1,1}+a_{1,2})(a_{2,1}+a_{2,2})=a_{1,1}a_{2,1}+a_{1,1}a_{2,2}+a_{1,2}a_{2,1}+a_{1,2}a_{2,2}$$ while $$sum_{2in{j_1,j_2}}a_{1,j_1}a_{2,j_2}=a_{1,1}a_{2,2}+a_{1,2}a_{2,1}+a_{1,2}a_{2,2}.$$ The term $a_{1,1}a_{2,1}$ is gone because $2notin{j_1,j_2}={1,1}$.
            – bof
            Nov 30 '18 at 6:57











          Your Answer





          StackExchange.ifUsing("editor", function () {
          return StackExchange.using("mathjaxEditing", function () {
          StackExchange.MarkdownEditor.creationCallbacks.add(function (editor, postfix) {
          StackExchange.mathjaxEditing.prepareWmdForMathJax(editor, postfix, [["$", "$"], ["\\(","\\)"]]);
          });
          });
          }, "mathjax-editing");

          StackExchange.ready(function() {
          var channelOptions = {
          tags: "".split(" "),
          id: "69"
          };
          initTagRenderer("".split(" "), "".split(" "), channelOptions);

          StackExchange.using("externalEditor", function() {
          // Have to fire editor after snippets, if snippets enabled
          if (StackExchange.settings.snippets.snippetsEnabled) {
          StackExchange.using("snippets", function() {
          createEditor();
          });
          }
          else {
          createEditor();
          }
          });

          function createEditor() {
          StackExchange.prepareEditor({
          heartbeatType: 'answer',
          autoActivateHeartbeat: false,
          convertImagesToLinks: true,
          noModals: true,
          showLowRepImageUploadWarning: true,
          reputationToPostImages: 10,
          bindNavPrevention: true,
          postfix: "",
          imageUploader: {
          brandingHtml: "Powered by u003ca class="icon-imgur-white" href="https://imgur.com/"u003eu003c/au003e",
          contentPolicyHtml: "User contributions licensed under u003ca href="https://creativecommons.org/licenses/by-sa/3.0/"u003ecc by-sa 3.0 with attribution requiredu003c/au003e u003ca href="https://stackoverflow.com/legal/content-policy"u003e(content policy)u003c/au003e",
          allowUrls: true
          },
          noCode: true, onDemand: true,
          discardSelector: ".discard-answer"
          ,immediatelyShowMarkdownHelp:true
          });


          }
          });














          draft saved

          draft discarded


















          StackExchange.ready(
          function () {
          StackExchange.openid.initPostLogin('.new-post-login', 'https%3a%2f%2fmath.stackexchange.com%2fquestions%2f3015646%2fproduce-formula%23new-answer', 'question_page');
          }
          );

          Post as a guest















          Required, but never shown

























          1 Answer
          1






          active

          oldest

          votes








          1 Answer
          1






          active

          oldest

          votes









          active

          oldest

          votes






          active

          oldest

          votes









          3














          $$a_{1,3}(a_{2,1}+a_{2, 2}+a_{2,3})(a_{3,1}+a_{3, 2}+a_{3,3})+a_{2,3}(a_{1,1}+a_{1, 2})(a_{3,1}+a_{3, 2}+a_{3,3})+a_{3,3}(a_{1,1}+a_{1, 2})(a_{2,1}+a_{2, 2})$$
          $$=(a_{1,1}+a_{1,2}+a_{1,3})(a_{2,1}+a_{2,2}+a_{2,3})(a_{3,1}+a_{3,2}+a_{3,3})-(a_{1,1}+a_{1,2})(a_{2,1}+a_{2,2})(a_{3,1}+a_{3,2})$$
          $$=prod_{i=1}^3sum_{j=1}^3a_{i,j}-prod_{i=1}^3sum_{j=1}^2a_{i,j}.$$
          More generally,
          $$sum_{i=1}^ra_{i,m}left(prod_{h=1}^{i-1}sum_{j=1}^{m-1}a_{h,j}right)left(prod_{h=i+1}^rsum_{j=1}^ma_{h,j}right)=prod_{i=1}^rsum_{j=1}^ma_{i,j}-prod_{i=1}^rsum_{j=1}^{m-1}a_{i,j}$$
          $$=sum_{min{j_1,j_2,dots,j_r}}a_{1,j_1}a_{2,j_2}cdots a_{r,j_r}.$$






          share|cite|improve this answer























          • Thank you for the answer. Is it possible to generalize it? I edited my question.
            – 1Spectre1
            Nov 27 '18 at 13:24






          • 1




            @1ENİGMA1 I cleaned up my answer.
            – bof
            Nov 28 '18 at 23:13






          • 1




            For example, $n!=prod_{i=1}^ni$ and in particular $0!=prod_{i=1}^0i=1$. Likewise, $x^n=prod_{i=1}^nx$, in particular, $x^0=prod_{i=1}^0x=1$.
            – bof
            Nov 29 '18 at 8:59








          • 1




            $sum_{min{j_1,j_2,dots,j_r}}a_{1,j_1}a_{2,j_2}cdots a_{r,j_r}$ is the sum of all products of the form $a_{1,j_1}a_{2,j_2}cdots a_{r,j_r}$ where at least one of the indices $j_1,j_2,dots,j_r$ is equal to $m$, or in other words, $m$ is an element of the set ${j_1,j_2,dots,j_r}$.
            – bof
            Nov 29 '18 at 9:04






          • 1




            $$prod_{i=1}^2sum_{j=1}^2a_{i,j}=(a_{1,1}+a_{1,2})(a_{2,1}+a_{2,2})=a_{1,1}a_{2,1}+a_{1,1}a_{2,2}+a_{1,2}a_{2,1}+a_{1,2}a_{2,2}$$ while $$sum_{2in{j_1,j_2}}a_{1,j_1}a_{2,j_2}=a_{1,1}a_{2,2}+a_{1,2}a_{2,1}+a_{1,2}a_{2,2}.$$ The term $a_{1,1}a_{2,1}$ is gone because $2notin{j_1,j_2}={1,1}$.
            – bof
            Nov 30 '18 at 6:57
















          3














          $$a_{1,3}(a_{2,1}+a_{2, 2}+a_{2,3})(a_{3,1}+a_{3, 2}+a_{3,3})+a_{2,3}(a_{1,1}+a_{1, 2})(a_{3,1}+a_{3, 2}+a_{3,3})+a_{3,3}(a_{1,1}+a_{1, 2})(a_{2,1}+a_{2, 2})$$
          $$=(a_{1,1}+a_{1,2}+a_{1,3})(a_{2,1}+a_{2,2}+a_{2,3})(a_{3,1}+a_{3,2}+a_{3,3})-(a_{1,1}+a_{1,2})(a_{2,1}+a_{2,2})(a_{3,1}+a_{3,2})$$
          $$=prod_{i=1}^3sum_{j=1}^3a_{i,j}-prod_{i=1}^3sum_{j=1}^2a_{i,j}.$$
          More generally,
          $$sum_{i=1}^ra_{i,m}left(prod_{h=1}^{i-1}sum_{j=1}^{m-1}a_{h,j}right)left(prod_{h=i+1}^rsum_{j=1}^ma_{h,j}right)=prod_{i=1}^rsum_{j=1}^ma_{i,j}-prod_{i=1}^rsum_{j=1}^{m-1}a_{i,j}$$
          $$=sum_{min{j_1,j_2,dots,j_r}}a_{1,j_1}a_{2,j_2}cdots a_{r,j_r}.$$






          share|cite|improve this answer























          • Thank you for the answer. Is it possible to generalize it? I edited my question.
            – 1Spectre1
            Nov 27 '18 at 13:24






          • 1




            @1ENİGMA1 I cleaned up my answer.
            – bof
            Nov 28 '18 at 23:13






          • 1




            For example, $n!=prod_{i=1}^ni$ and in particular $0!=prod_{i=1}^0i=1$. Likewise, $x^n=prod_{i=1}^nx$, in particular, $x^0=prod_{i=1}^0x=1$.
            – bof
            Nov 29 '18 at 8:59








          • 1




            $sum_{min{j_1,j_2,dots,j_r}}a_{1,j_1}a_{2,j_2}cdots a_{r,j_r}$ is the sum of all products of the form $a_{1,j_1}a_{2,j_2}cdots a_{r,j_r}$ where at least one of the indices $j_1,j_2,dots,j_r$ is equal to $m$, or in other words, $m$ is an element of the set ${j_1,j_2,dots,j_r}$.
            – bof
            Nov 29 '18 at 9:04






          • 1




            $$prod_{i=1}^2sum_{j=1}^2a_{i,j}=(a_{1,1}+a_{1,2})(a_{2,1}+a_{2,2})=a_{1,1}a_{2,1}+a_{1,1}a_{2,2}+a_{1,2}a_{2,1}+a_{1,2}a_{2,2}$$ while $$sum_{2in{j_1,j_2}}a_{1,j_1}a_{2,j_2}=a_{1,1}a_{2,2}+a_{1,2}a_{2,1}+a_{1,2}a_{2,2}.$$ The term $a_{1,1}a_{2,1}$ is gone because $2notin{j_1,j_2}={1,1}$.
            – bof
            Nov 30 '18 at 6:57














          3












          3








          3






          $$a_{1,3}(a_{2,1}+a_{2, 2}+a_{2,3})(a_{3,1}+a_{3, 2}+a_{3,3})+a_{2,3}(a_{1,1}+a_{1, 2})(a_{3,1}+a_{3, 2}+a_{3,3})+a_{3,3}(a_{1,1}+a_{1, 2})(a_{2,1}+a_{2, 2})$$
          $$=(a_{1,1}+a_{1,2}+a_{1,3})(a_{2,1}+a_{2,2}+a_{2,3})(a_{3,1}+a_{3,2}+a_{3,3})-(a_{1,1}+a_{1,2})(a_{2,1}+a_{2,2})(a_{3,1}+a_{3,2})$$
          $$=prod_{i=1}^3sum_{j=1}^3a_{i,j}-prod_{i=1}^3sum_{j=1}^2a_{i,j}.$$
          More generally,
          $$sum_{i=1}^ra_{i,m}left(prod_{h=1}^{i-1}sum_{j=1}^{m-1}a_{h,j}right)left(prod_{h=i+1}^rsum_{j=1}^ma_{h,j}right)=prod_{i=1}^rsum_{j=1}^ma_{i,j}-prod_{i=1}^rsum_{j=1}^{m-1}a_{i,j}$$
          $$=sum_{min{j_1,j_2,dots,j_r}}a_{1,j_1}a_{2,j_2}cdots a_{r,j_r}.$$






          share|cite|improve this answer














          $$a_{1,3}(a_{2,1}+a_{2, 2}+a_{2,3})(a_{3,1}+a_{3, 2}+a_{3,3})+a_{2,3}(a_{1,1}+a_{1, 2})(a_{3,1}+a_{3, 2}+a_{3,3})+a_{3,3}(a_{1,1}+a_{1, 2})(a_{2,1}+a_{2, 2})$$
          $$=(a_{1,1}+a_{1,2}+a_{1,3})(a_{2,1}+a_{2,2}+a_{2,3})(a_{3,1}+a_{3,2}+a_{3,3})-(a_{1,1}+a_{1,2})(a_{2,1}+a_{2,2})(a_{3,1}+a_{3,2})$$
          $$=prod_{i=1}^3sum_{j=1}^3a_{i,j}-prod_{i=1}^3sum_{j=1}^2a_{i,j}.$$
          More generally,
          $$sum_{i=1}^ra_{i,m}left(prod_{h=1}^{i-1}sum_{j=1}^{m-1}a_{h,j}right)left(prod_{h=i+1}^rsum_{j=1}^ma_{h,j}right)=prod_{i=1}^rsum_{j=1}^ma_{i,j}-prod_{i=1}^rsum_{j=1}^{m-1}a_{i,j}$$
          $$=sum_{min{j_1,j_2,dots,j_r}}a_{1,j_1}a_{2,j_2}cdots a_{r,j_r}.$$







          share|cite|improve this answer














          share|cite|improve this answer



          share|cite|improve this answer








          edited Nov 28 '18 at 23:11

























          answered Nov 27 '18 at 12:27









          bofbof

          50.5k457119




          50.5k457119












          • Thank you for the answer. Is it possible to generalize it? I edited my question.
            – 1Spectre1
            Nov 27 '18 at 13:24






          • 1




            @1ENİGMA1 I cleaned up my answer.
            – bof
            Nov 28 '18 at 23:13






          • 1




            For example, $n!=prod_{i=1}^ni$ and in particular $0!=prod_{i=1}^0i=1$. Likewise, $x^n=prod_{i=1}^nx$, in particular, $x^0=prod_{i=1}^0x=1$.
            – bof
            Nov 29 '18 at 8:59








          • 1




            $sum_{min{j_1,j_2,dots,j_r}}a_{1,j_1}a_{2,j_2}cdots a_{r,j_r}$ is the sum of all products of the form $a_{1,j_1}a_{2,j_2}cdots a_{r,j_r}$ where at least one of the indices $j_1,j_2,dots,j_r$ is equal to $m$, or in other words, $m$ is an element of the set ${j_1,j_2,dots,j_r}$.
            – bof
            Nov 29 '18 at 9:04






          • 1




            $$prod_{i=1}^2sum_{j=1}^2a_{i,j}=(a_{1,1}+a_{1,2})(a_{2,1}+a_{2,2})=a_{1,1}a_{2,1}+a_{1,1}a_{2,2}+a_{1,2}a_{2,1}+a_{1,2}a_{2,2}$$ while $$sum_{2in{j_1,j_2}}a_{1,j_1}a_{2,j_2}=a_{1,1}a_{2,2}+a_{1,2}a_{2,1}+a_{1,2}a_{2,2}.$$ The term $a_{1,1}a_{2,1}$ is gone because $2notin{j_1,j_2}={1,1}$.
            – bof
            Nov 30 '18 at 6:57


















          • Thank you for the answer. Is it possible to generalize it? I edited my question.
            – 1Spectre1
            Nov 27 '18 at 13:24






          • 1




            @1ENİGMA1 I cleaned up my answer.
            – bof
            Nov 28 '18 at 23:13






          • 1




            For example, $n!=prod_{i=1}^ni$ and in particular $0!=prod_{i=1}^0i=1$. Likewise, $x^n=prod_{i=1}^nx$, in particular, $x^0=prod_{i=1}^0x=1$.
            – bof
            Nov 29 '18 at 8:59








          • 1




            $sum_{min{j_1,j_2,dots,j_r}}a_{1,j_1}a_{2,j_2}cdots a_{r,j_r}$ is the sum of all products of the form $a_{1,j_1}a_{2,j_2}cdots a_{r,j_r}$ where at least one of the indices $j_1,j_2,dots,j_r$ is equal to $m$, or in other words, $m$ is an element of the set ${j_1,j_2,dots,j_r}$.
            – bof
            Nov 29 '18 at 9:04






          • 1




            $$prod_{i=1}^2sum_{j=1}^2a_{i,j}=(a_{1,1}+a_{1,2})(a_{2,1}+a_{2,2})=a_{1,1}a_{2,1}+a_{1,1}a_{2,2}+a_{1,2}a_{2,1}+a_{1,2}a_{2,2}$$ while $$sum_{2in{j_1,j_2}}a_{1,j_1}a_{2,j_2}=a_{1,1}a_{2,2}+a_{1,2}a_{2,1}+a_{1,2}a_{2,2}.$$ The term $a_{1,1}a_{2,1}$ is gone because $2notin{j_1,j_2}={1,1}$.
            – bof
            Nov 30 '18 at 6:57
















          Thank you for the answer. Is it possible to generalize it? I edited my question.
          – 1Spectre1
          Nov 27 '18 at 13:24




          Thank you for the answer. Is it possible to generalize it? I edited my question.
          – 1Spectre1
          Nov 27 '18 at 13:24




          1




          1




          @1ENİGMA1 I cleaned up my answer.
          – bof
          Nov 28 '18 at 23:13




          @1ENİGMA1 I cleaned up my answer.
          – bof
          Nov 28 '18 at 23:13




          1




          1




          For example, $n!=prod_{i=1}^ni$ and in particular $0!=prod_{i=1}^0i=1$. Likewise, $x^n=prod_{i=1}^nx$, in particular, $x^0=prod_{i=1}^0x=1$.
          – bof
          Nov 29 '18 at 8:59






          For example, $n!=prod_{i=1}^ni$ and in particular $0!=prod_{i=1}^0i=1$. Likewise, $x^n=prod_{i=1}^nx$, in particular, $x^0=prod_{i=1}^0x=1$.
          – bof
          Nov 29 '18 at 8:59






          1




          1




          $sum_{min{j_1,j_2,dots,j_r}}a_{1,j_1}a_{2,j_2}cdots a_{r,j_r}$ is the sum of all products of the form $a_{1,j_1}a_{2,j_2}cdots a_{r,j_r}$ where at least one of the indices $j_1,j_2,dots,j_r$ is equal to $m$, or in other words, $m$ is an element of the set ${j_1,j_2,dots,j_r}$.
          – bof
          Nov 29 '18 at 9:04




          $sum_{min{j_1,j_2,dots,j_r}}a_{1,j_1}a_{2,j_2}cdots a_{r,j_r}$ is the sum of all products of the form $a_{1,j_1}a_{2,j_2}cdots a_{r,j_r}$ where at least one of the indices $j_1,j_2,dots,j_r$ is equal to $m$, or in other words, $m$ is an element of the set ${j_1,j_2,dots,j_r}$.
          – bof
          Nov 29 '18 at 9:04




          1




          1




          $$prod_{i=1}^2sum_{j=1}^2a_{i,j}=(a_{1,1}+a_{1,2})(a_{2,1}+a_{2,2})=a_{1,1}a_{2,1}+a_{1,1}a_{2,2}+a_{1,2}a_{2,1}+a_{1,2}a_{2,2}$$ while $$sum_{2in{j_1,j_2}}a_{1,j_1}a_{2,j_2}=a_{1,1}a_{2,2}+a_{1,2}a_{2,1}+a_{1,2}a_{2,2}.$$ The term $a_{1,1}a_{2,1}$ is gone because $2notin{j_1,j_2}={1,1}$.
          – bof
          Nov 30 '18 at 6:57




          $$prod_{i=1}^2sum_{j=1}^2a_{i,j}=(a_{1,1}+a_{1,2})(a_{2,1}+a_{2,2})=a_{1,1}a_{2,1}+a_{1,1}a_{2,2}+a_{1,2}a_{2,1}+a_{1,2}a_{2,2}$$ while $$sum_{2in{j_1,j_2}}a_{1,j_1}a_{2,j_2}=a_{1,1}a_{2,2}+a_{1,2}a_{2,1}+a_{1,2}a_{2,2}.$$ The term $a_{1,1}a_{2,1}$ is gone because $2notin{j_1,j_2}={1,1}$.
          – bof
          Nov 30 '18 at 6:57


















          draft saved

          draft discarded




















































          Thanks for contributing an answer to Mathematics Stack Exchange!


          • Please be sure to answer the question. Provide details and share your research!

          But avoid



          • Asking for help, clarification, or responding to other answers.

          • Making statements based on opinion; back them up with references or personal experience.


          Use MathJax to format equations. MathJax reference.


          To learn more, see our tips on writing great answers.





          Some of your past answers have not been well-received, and you're in danger of being blocked from answering.


          Please pay close attention to the following guidance:


          • Please be sure to answer the question. Provide details and share your research!

          But avoid



          • Asking for help, clarification, or responding to other answers.

          • Making statements based on opinion; back them up with references or personal experience.


          To learn more, see our tips on writing great answers.




          draft saved


          draft discarded














          StackExchange.ready(
          function () {
          StackExchange.openid.initPostLogin('.new-post-login', 'https%3a%2f%2fmath.stackexchange.com%2fquestions%2f3015646%2fproduce-formula%23new-answer', 'question_page');
          }
          );

          Post as a guest















          Required, but never shown





















































          Required, but never shown














          Required, but never shown












          Required, but never shown







          Required, but never shown

































          Required, but never shown














          Required, but never shown












          Required, but never shown







          Required, but never shown







          Popular posts from this blog

          Plaza Victoria

          In PowerPoint, is there a keyboard shortcut for bulleted / numbered list?

          How to put 3 figures in Latex with 2 figures side by side and 1 below these side by side images but in...